Difference between revisions of "2013 AIME I Problems/Problem 8"
(→Solution) |
|||
Line 1: | Line 1: | ||
== Problem 8 == | == Problem 8 == | ||
− | The domain of the function f(x) = arcsin(log<math>_{m}</math>( | + | The domain of the function f(x) = arcsin(log<math>_{m}</math>(nx)) is a closed interval of length <math>\frac{1}{2013}</math> , where <math>m</math> and <math>n</math> are positive integers and <math>m>1</math>. Find the remainder when the smallest possible sum <math>m+n</math> is divided by 1000. |
== Solution == | == Solution == | ||
− | The domain of the arcsin function is [-1, 1], so | + | The domain of the arcsin function is <math>[-1, 1]</math>, so <math>-1 \le</math> log_{m}(nx) \le 1<math>. |
− | <math>\frac{1}{m} \le nx \le m< | + | </math>\frac{1}{m} \le nx \le m<math> |
− | <math>\frac{1}{mn} \le x \le \frac{m}{n}< | + | </math>\frac{1}{mn} \le x \le \frac{m}{n}<math> |
− | <math>\frac{m}{n} - \frac{1}{mn} = \frac{1}{2013}< | + | </math>\frac{m}{n} - \frac{1}{mn} = \frac{1}{2013}<math> |
− | <math>n = 2013m - \frac{2013}{m}< | + | </math>n = 2013m - \frac{2013}{m}<math> |
− | For | + | For </math>n<math> to be an integer, </math>m<math> must divide </math>2013<math>, and </math>m > 1<math>. To minimize </math>n<math>, </math>m<math> should be as small as possible because increasing </math>m<math> will decrease </math>\frac{2013}{m}<math> , the amount you are subtracting, and increase </math>2013m<math> , the amount you are adding; this also leads to a small </math>m<math> which clearly minimizes '</math>m+n<math>. |
− | We let | + | We let </math>m<math> equal 3, the smallest factor of </math>2013<math> that isn't </math>1<math>.. </math>n = 2013*3 - \frac{2013}{3} = 6039 - 671 = 5368<math> |
− | <math>m + n = 5371 | + | </math>m + n = 5371<math>, so the answer is </math>\boxed{371}$. |
Revision as of 17:19, 16 March 2013
Problem 8
The domain of the function f(x) = arcsin(log(nx)) is a closed interval of length , where and are positive integers and . Find the remainder when the smallest possible sum is divided by 1000.
Solution
The domain of the arcsin function is , so log_{m}(nx) \le 1\frac{1}{m} \le nx \le m$$ (Error compiling LaTeX. Unknown error_msg)\frac{1}{mn} \le x \le \frac{m}{n}$$ (Error compiling LaTeX. Unknown error_msg)\frac{m}{n} - \frac{1}{mn} = \frac{1}{2013}$$ (Error compiling LaTeX. Unknown error_msg)n = 2013m - \frac{2013}{m}nm2013m > 1nmm\frac{2013}{m}2013mmm+n$.
We let$ (Error compiling LaTeX. Unknown error_msg)m20131n = 2013*3 - \frac{2013}{3} = 6039 - 671 = 5368$$ (Error compiling LaTeX. Unknown error_msg)m + n = 5371\boxed{371}$.